2005 iTest Problems/Problem 19

Revision as of 10:03, 14 October 2025 by Mathloveryeah (talk | contribs) (Created page with "==Problem== Find the amplitude of <math>y = 4 \sin (x) + 3 \cos (x)</math>. ==Solution 1== By the Cauchy-Schwarz Inequality, <math>(a \cos \theta + b \sin \theta)^2 \le (a^2...")
(diff) ← Older revision | Latest revision (diff) | Newer revision → (diff)

Problem

Find the amplitude of $y = 4 \sin (x) + 3 \cos (x)$.

Solution 1

By the Cauchy-Schwarz Inequality, $(a \cos \theta + b \sin \theta)^2 \le (a^2 + b^2)(\cos^2 \theta + \sin^2 \theta) = a^2 + b^2$, so $a \cos \theta +b \sin \theta \le \sqrt{a^2+b^2}$. If $a=b=0$, then the expression is equal to zero for any angle. Otherwise, we can construct a right triangle with an angle measuring $\theta$ with side lengths $a,b$ and $\sqrt{a^2+b^2}$. Evaluating the trig functions for this angle yields $\cos \theta=\frac{a}{\sqrt{a^2+b^2}}$ and $\sin \theta=\frac{b}{\sqrt{a^2}{b^2}}$, and multiplying the cosine by $a$ and the since by $b$ and adding yields that $a\cos \theta +b \sin \theta=\sqrt{a^2+b^2}$. Plugging in 3 and 4, we find that the maximum value is $5$. Since this is a trig function with midline $y=0$, its amplitude is equal to its maximum value, which is $\boxed{5}$.

See Also

2005 iTest (Problems, Answer Key)
Preceded by:
Problem 15
Followed by:
Problem 17
1 2 3 4 5 6 7 8 9 10 11 12 13 14 15 16 17 18 19 20 21 22 23 24 25 26 27 28 29 30 31 32 33 34 35 36 37 38 39 40 41 42 43 44 45 46 47 48 49 50 51 52 53 54 55 56 57 58 59 60 TB1 TB2 TB3 TB4